An analysis of a rotating rod

Consider a uniform rod of mass M M and length L , L, free to rotate around a frictionless axis passing through its center and going into the page. Initially, the rod is stationary in the horizontal position, as shown in the diagram below.

Now, a small bullet of mass m m moving with velocity v v hits the rod at its extreme end and sticks to it. The system rotates vertically through some angle θ \theta before it momentarily comes to rest. If this angle can be expressed (in degrees) as θ = α + arcsin ( β m v 2 ( M + γ m ) g L ) , \theta = \alpha + \arcsin \left(\frac {\beta mv^2}{( M+\gamma m)gL}\right), where g g denotes the gravitational acceleration and α \alpha , β \beta , and γ \gamma are positive integer constants with α \alpha in degrees, then find the value of α + β + γ \alpha + \beta + \gamma .


The answer is 186.

This section requires Javascript.
You are seeing this because something didn't load right. We suggest you, (a) try refreshing the page, (b) enabling javascript if it is disabled on your browser and, finally, (c) loading the non-javascript version of this page . We're sorry about the hassle.

3 solutions

Steven Chase
Sep 4, 2017

Relevant wiki: Conservation of Angular Momentum

Conservation of angular momentum ( ω \omega is the angular frequency immediately after the collision):

( L 2 ) m v = ( L 2 ) 2 m ω + M L 2 12 ω ω = 2 m v m L + M L 3 \big(\frac{L}{2}\big) m v = \big(\frac{L}{2}\big)^2 m \omega + \frac{M L^2}{12} \omega \\ \implies \omega = \frac{2mv}{mL + \frac{ML}{3}}

The gravitational potential energy of the rod is constant, regardless of its orientation. Therefore, the system comes to rest when the system turns around 180 degrees, and then the bead rises high enough to transform all of the initial kinetic energy into gravitational potential energy relative to the start height. In the equation below, the left side gives the initial kinetic energy, and the right side is the potential energy relative to the start height.

1 2 m ( L 2 ) 2 ω 2 + 1 2 M L 2 12 ω 2 = m g L 2 s i n θ ( 1 2 m ( L 2 ) 2 + 1 2 M L 2 12 ) 4 m 2 v 2 ( m L + M L 3 ) 2 = m g L 2 s i n θ ( m L 4 + M L 12 ) 4 m 2 v 2 ( m L + M L 3 ) 2 = m g s i n θ ( m L + M L 3 ) m 2 v 2 ( m L + M L 3 ) 2 = m g s i n θ m 2 v 2 ( m L + M L 3 ) = m g s i n θ m v 2 g L ( m + M 3 ) = 3 m v 2 g L ( 3 m + M ) = s i n θ \frac{1}{2} m \big(\frac{L}{2}\big)^2 \omega^2 + \frac{1}{2} \frac{M L^2}{12} \omega^2 = mg \frac{L}{2} sin\theta \\ (\frac{1}{2} m \big(\frac{L}{2}\big)^2 + \frac{1}{2} \frac{M L^2}{12}) \frac{4 m^2 v^2}{(mL + \frac{ML}{3})^2} = mg \frac{L}{2} sin\theta \\ (\frac{mL}{4} + \frac{M L}{12}) \frac{4 m^2 v^2}{(mL + \frac{ML}{3})^2} = mg sin\theta \\ (mL + \frac{M L}{3}) \frac{m^2 v^2}{(mL + \frac{ML}{3})^2} = mg sin\theta \\ \frac{m^2 v^2}{(mL + \frac{ML}{3})} = mg sin\theta \\ \frac{m v^2}{gL(m + \frac{M}{3})} = \frac{3m v^2}{gL(3m + M)} = sin\theta

The full angle is therefore:

180 + a s i n ( 3 m v 2 g L ( 3 m + M ) ) 180 + asin\big(\frac{3m v^2}{gL(3m + M)}\big)

Very nice solution.

Ayon Ghosh - 3 years, 9 months ago

Thanks. It was a fun problem

Steven Chase - 3 years, 9 months ago
Ayon Ghosh
Sep 3, 2017

Relevant wiki: Applying Angular Kinematic Equations

In this solution I have used energy considerations and integral calculus.

First applying L a x i s L_{axis} = = I a x i s , n e w I_{axis,new} ω i n i t i a l ω_{initial} we get ω i n i t i a l ω_{initial} .

Now,The basic gist is that as the rod rotates there is a net external torque due to the "extra" mass of the bullet on one side acts and accelerates the rod from 0 0 degrees to 90 90 degrees.But the torque acting is variable and is dependent on the angle the rod has turned at a particular instant.So to find the total work done i have simply integrated the torque acting to avoid complex calculations if we would have had applied torque = I a x i s I_{axis} α \alpha

Now what happens is that from 90 90 degrees to 180 180 degrees that same torque decelerates the rod.But then from 180 180 degrees to 270 270 degrees the torque again opposes the motion thus sometime between 180 180 and 270 270 degrees,the rod must have come to rest.So, support then oppose then again oppose.This causes it to finally stop.For rigour I have shown that the work done from 0 0 degrees to 90 90 degrees if work done is W W then from 90 90 degrees to 180 180 degrees work done is W -W .Hence net work done from 0 0 degrees to 180 180 degrees is 0 0 .Thus it is moving with the same angular velocity that it has initially acquired.But as stated earlier now from 180 180 degrees to 270 270 degrees the torque is decelerating so it will finally stop somewhere along the interval [ 180 , 270 ] [180,270] .Our aim is to find this angle and add it to the original 180 180 degrees.already we have got α \alpha = = 180 180 degrees.

This is rigorously shown below:

So the answer is 180 + 3 + 3 180 + 3 + 3 = = 186 186


Question is original and made by me.Hope you liked it.

Bhai, You must tell that α \alpha is an angle. I think it must be mentioned. Thanks

Ayaen Shukla - 3 years, 9 months ago

Log in to reply

@Ayon Ghosh , Same thoughts brother!

Md Zuhair - 3 years, 9 months ago

@Ayaen Shukla , @Md Zuhair guys i have done the required editing.

Ayon Ghosh - 3 years, 9 months ago

Log in to reply

now it seems ok. :). Keep posting nice rotation problems. I like them. If u want tough rotation problems u can try anikets mechanics challenges

Md Zuhair - 3 years, 9 months ago

@Md Zuhair Yes i have tried some of aniket 's challenges.they are really good and he is also very intelligent from my state he is jee topper.

Ayon Ghosh - 3 years, 9 months ago

Log in to reply

Have u given prmo? If yes, how many correct?

Md Zuhair - 3 years, 9 months ago

@Md Zuhair 11 (including bonus).How about you ?

Ayon Ghosh - 3 years, 9 months ago

Log in to reply

Same here :(. I got 2 incorrect marking :(. I am quite sad about it. and 2 silly :(.

Md Zuhair - 3 years, 9 months ago

Including all the three bonuses as per PRMO official answer key?

PRE RMO OFFICIAL ANSWER KEYS

Md Zuhair - 3 years, 9 months ago

You all are struggling with PRERMO? Hmm, Nice exam. There was an hon'ble mention for me in INMO.

Ayaen Shukla - 3 years, 9 months ago

Log in to reply

Wow, a IIT'ian u are! You are great!

Md Zuhair - 3 years, 9 months ago

@Md Zuhair .

Ayon Ghosh - 3 years, 9 months ago

no problem there's rmo as well prepare stronger for itand try to aim for imotc/inmo.btw its good to see that you are preparing for math olympiads and not running behind jee in class 11.Everyone here in bhilai in 11th is after jee its like the math olympiad thing lasts only till 10th.I guess people don't understand the beauty of math olympiad problems.

Ayon Ghosh - 3 years, 9 months ago

Log in to reply

Oh, I see....

Md Zuhair - 3 years, 9 months ago

@Ayaen Shukla honorable mention means you made it to IMOTC ?

Ayon Ghosh - 3 years, 9 months ago

Log in to reply

Ah... IMOTC is a nice place. :)

Ayaen Shukla - 3 years, 9 months ago

Poorly worded question

Mohd Siddiqui - 3 years, 9 months ago

Log in to reply

Is there something wrong with the wording? Do you have any suggestions on how to improve it?

Agnishom Chattopadhyay - 3 years, 8 months ago

@Mohd Siddiqui i have not worded it the brillant officials have.

Ayon Ghosh - 3 years, 9 months ago

Log in to reply

The wordings seem completely okay to me.

Md Zuhair - 3 years, 9 months ago

Fun problem, but yes the wording made it look that the bar was in an horizontal plane initially, maybe it was due to saying the "axis going to the page"+"the rod is horizontal position", but after rereading and some assumptions, it was clear what was supposed to be asked.

Luis Enrique Gomez Sanchez - 3 years, 7 months ago
Arjen Vreugdenhil
Sep 13, 2017

Relevant wiki: Rotational Kinetic Energy - Problem Solving

The rod + small bullet have rotational inertia I = 1 12 M 2 + m ( 1 2 ) 2 = 1 12 ( M + 3 m ) 2 . I = \tfrac1{12}M\ell^2 + m(\tfrac12\ell)^2 = \tfrac1{12}(M + 3m)\ell^2. The initial angular momentum of the system comes from the bullet, and is equal to L = m v r = 1 2 m v . L = mvr = \tfrac12mv\ell. This momentum is conserved; it is transferred to the rod + bullet system. If we wanted, we could solve for the rotational speed ω = L / I \omega = L/I . However, this is not necessary.

Consider now the conservation of energy during the swing. The rod's potential energy is constant, so we only consider the bullet's PE, which is equal to m g y mgy with y = 1 2 sin θ y = -\tfrac12\ell\sin\theta . We are interested in the situation where the KE of the system becomes zero. Thus 1 2 m g sin θ = 1 2 I ω 2 = 1 2 L 2 I ; -\tfrac12 mg\ell\sin\theta = \tfrac12I\omega^2 = \tfrac12\frac{L^2}I; sin θ = L 2 m g I = ( 1 2 m v ) 2 m g 1 12 ( M + 3 m ) 2 = 3 m v 2 ( M + 3 m ) g . \sin\theta = -\frac{L^2}{mg\ell I} = -\frac{(\tfrac12mv\ell)^2}{mg\ell\cdot \tfrac1{12}(M + 3m)\ell^2} = -\frac{3mv^2}{(M+3m)g\ell}. Since this sin θ < 0 \sin\theta < 0 and θ > 0 \theta > 0 , the first solution will have θ \theta in the third quadrant; so θ = 18 0 + arc sin 3 m v 2 ( M + 3 m ) g . \theta = 180^\circ + \text{arc sin}\ \frac{3mv^2}{(M+3m)g\ell}. We submit α + β + γ = 180 + 3 + 3 = 186 . \alpha + \beta + \gamma = 180 + 3 + 3 = \boxed{186}.

@Arjen Vreugdenhil wow that 's the best and the shortest solution to this problem ! Hats off !

Ayon Ghosh - 3 years, 9 months ago

0 pending reports

×

Problem Loading...

Note Loading...

Set Loading...